Proof that a recursive sequence converges

  • Thread starter Thread starter Mr Davis 97
  • Start date Start date
  • Tags Tags
    Proof Sequence
Click For Summary
The recursive sequence defined by t_{n+1} = (1 - 1/(4n^2)) t_n with t_1 = 1 is shown to converge by proving it is bounded below and decreasing. The base case establishes that t_1 is non-negative, and induction confirms that if t_k is non-negative, then t_{k+1} is also non-negative. The sequence is demonstrated to be decreasing by showing that t_{k+2} is less than or equal to t_{k+1}, completing the induction. By the Monotone Convergence Theorem, the sequence converges, with additional insights suggesting geometrically fast convergence. Thus, the sequence converges to a limit as n approaches infinity.
Mr Davis 97
Messages
1,461
Reaction score
44

Homework Statement


Prove that ##\displaystyle t_{n+1} = (1 - \frac{1}{4n^2}) t_n## where ##t_1=1## converges.

Homework Equations

The Attempt at a Solution


First, we must prove that the sequence is bounded below. We will prove that it is bounded below by 0. ##t_1 = 1 \ge 0##, so the base case holds. Now, suppose that ##t_k \ge 0##. Then ##t_{k+1} = (1 - \frac{1}{4k^2}) t_k \ge (1 - \frac{1}{4k^2}) (0) = 0##. So the induction is complete, and the sequence is bounded below by 0.

Now we must show that the sequence is decreasing. ##t_2 = 15/16 \le 1 = t_1##, so the base case holds. Now, suppose that ##t_{k+1} \le t_k##. Then ##t_{k+2} = (1 - \frac{1}{4(k+1)^2})t_{k+1} \le t_{k+1}##. So the induction is complete, and the sequence is decreasing.

By the Monotone Convergence Theorem, the sequence converges.
 
Physics news on Phys.org
Seems ok, btw you can actually solve this recursive equation by iteration, and see that it's decreasing explicitly.
 
Mr Davis 97 said:

Homework Statement


Prove that ##\displaystyle t_{n+1} = (1 - \frac{1}{4n^2}) t_n## where ##t_1=1## converges.

Homework Equations

The Attempt at a Solution


First, we must prove that the sequence is bounded below. We will prove that it is bounded below by 0. ##t_1 = 1 \ge 0##, so the base case holds. Now, suppose that ##t_k \ge 0##. Then ##t_{k+1} = (1 - \frac{1}{4k^2}) t_k \ge (1 - \frac{1}{4k^2}) (0) = 0##. So the induction is complete, and the sequence is bounded below by 0.

Now we must show that the sequence is decreasing. ##t_2 = 15/16 \le 1 = t_1##, so the base case holds. Now, suppose that ##t_{k+1} \le t_k##. Then ##t_{k+2} = (1 - \frac{1}{4(k+1)^2})t_{k+1} \le t_{k+1}##. So the induction is complete, and the sequence is decreasing.

By the Monotone Convergence Theorem, the sequence converges.

Another, perhaps easier way is to note that
$$|t_{n+1}-t_n| = \frac{1}{4n^2} |t_n| \leq \frac{1}{4} |t_n|,$$
so we have (at least) geometrically fast convergence.
 
One question. In showing that the sequence decreases, why didn't I have to use the inductive hypothesis?
 
Mr Davis 97 said:
One question. In showing that the sequence decreases, why didn't I have to use the inductive hypothesis?
$$ \frac{t_{n+1}}{t_n }= 1 - \frac{1}{4n^2} ,$$
so if you know that ##t_n > 0## then you have ##0 < t_{n+1}/t_n < 1,## hence a positive strictly decreasing sequence.
 
For n \geq 1, it follows from <br /> \frac34 \leq 1 - \frac{1}{4n^2} &lt; 1 that <br /> 0 &lt; |t_{n+1}| = \left|1 - \frac{1}{4n^2}\right||t_n| &lt; |t_n| and so |t_n| is strictly decreasing. Since each t_n is by construction a product of strictly positive reals we can immediately remove the absolute value signs.
 
Question: A clock's minute hand has length 4 and its hour hand has length 3. What is the distance between the tips at the moment when it is increasing most rapidly?(Putnam Exam Question) Answer: Making assumption that both the hands moves at constant angular velocities, the answer is ## \sqrt{7} .## But don't you think this assumption is somewhat doubtful and wrong?

Similar threads

  • · Replies 7 ·
Replies
7
Views
3K
  • · Replies 3 ·
Replies
3
Views
1K
  • · Replies 11 ·
Replies
11
Views
2K
  • · Replies 2 ·
Replies
2
Views
2K
Replies
15
Views
2K
  • · Replies 13 ·
Replies
13
Views
2K
  • · Replies 1 ·
Replies
1
Views
2K
Replies
4
Views
2K
Replies
3
Views
2K
Replies
4
Views
2K